Logo Icon

Laws of Torts Notes [ Download PDF Free ]

Author : Yogricha

Updated On : June 25, 2024

SHARE

Overview: The Law of Torts originated within the realm of common law, while Breach of Trust cases were typically addressed in the Chancery courts. Furthermore, the Law of Torts is not considered a subset of property law, whereas the Law of Trusts can be and is classified as a branch of property law.

In judiciary examinations like Uttar Pradesh Judiciary and other Judiciary Exams tort is an important subject that is asked in Prelims or Mains or both the satges of examination. If the state you are targetting has Law of Torts  In this Blog we will cover:

  • Imprortant topics of Tort
  • Practice Questions for Tort
  • Free notes for Judiciary preparation

Download FREE Study Material for Judiciary Exam by Judiciary Gold

Introduction to Law of Torts

The term 'Torts' finds its etymological roots in the Latin word "tortrum," signifying "to twist," while its French counterpart directly translates to a 'wrong.' In essence, a tort is a civil wrong, a private transgression that does not fall under the categories of Breach of Contract or Breach of Trust. To put it simply, any civil wrongdoing that cannot be classified as either a 'Breach of Contract' or a 'Breach of Trust' constitutes a tort.

Attempting to confine the concept of torts within a single, satisfactory definition is akin to trying to confine it within a rigid framework from which it perpetually seeks to break free.

Read about: Upcoming Judiciary Exams

A wrongful act can take the form of either a Public Wrong (a crime) or a Private Wrong (a civil wrong). Civil wrongs are further subdivided into three categories:

i. Breach of Contract ii. Breach of Trust iii. Tort

To establish whether a civil wrong qualifies as a tort, it must first be determined that the wrongdoing does not fall within the purview of the first two categories—Breach of Contract or Breach of Trust. If it does not fit into either of these categories, it is categorized as a tort. Consequently, the determination of whether an act constitutes a tort relies on a negative definition, employing a method of elimination.

The person committing a tort is referred to as a 'Tort-feasor.'

Tort stands as the only branch of law that remains un-codified, drawing its origins from the rulings of the House of Lords in England. The development and creation of tort law are shaped by court decisions, as it is not enshrined within a specific legislative act or statute.

Take a class: POCSO Act for Judiciary

A tort is committed whenever someone's act or omission violates a legal right vested in another person. This branch of law holds great significance in common law countries like India.

Essential Elements of Tort: To constitute an actionable tort, certain essential elements or prerequisites must be present, including:

I. Legal Injury (Injuria) – Legal injury signifies the violation of a legal right held by an individual.

II. Wrongful Act or Omission – The infliction of legal injury must stem from an individual's wrongful action or failure to act (omission). A wrongful act or omission arises when a person has a duty of care and breaches that duty, leading to legal injury to another party.

Download Free Notes: POCSO for Judiciary Examination

III. Availability of Legal Remedy – In order for a tort to be considered actionable, it is imperative that the wrongful act or omission falls within a category that permits the initiation of a civil lawsuit seeking damages (compensation).

The Two Fundamental Principles (Maxims) of Tort Law:

Tort law rests upon two fundamental maxims that are crucial for comprehending the essence of torts and how a tort becomes actionable:

  1. Injuria Sine Damnum: This maxim literally translates to 'Legal Injury (Injuria) without (Sine) Damage/Loss (Damnum).' In essence, Injuria Sine Damnum encompasses situations in which a person's legal rights are violated, yet no actual loss or damage is suffered as a consequence. Cases falling under 'Injuria Sine Damnum' constitute actionable torts. Therefore, the individual who experiences a legal injury is entitled to seek damages (compensation) from the court.

    Example: • A, a 21-year-old Indian citizen, was forcibly prevented by B from casting his vote in the General elections of his constituency. However, when the election results were announced, the candidate whom A intended to vote for emerged as the winner. • In this scenario, A's 'right to vote' (a legal right) was infringed upon by B's actions, but A did not endure any tangible loss since his preferred candidate won. Consequently, there is a Legal Injury (Injuria) without any accompanying damage or loss (Damnum). • This is a classic illustration of Injuria Sine Damnum, making it an actionable tort.

  2. Damnum Sine Injuria: This maxim literally means 'Damage (Damnum) without (Sine) a legal injury (Injuria).' Cases falling under this category do not qualify as actionable torts, and the plaintiff is not entitled to damages (compensation) even if they have incurred losses. This is because the key element required for tortious liability to arise is the presence of 'Legal Injury,' which is absent in such cases.

    Example: • A owns a bakery shop, and B is his employee. A enjoys a complete monopoly in the market. After some time, B opens a shop in A's vicinity, leading to competition in the market and a division of customers, resulting in significant financial losses for A. • In this scenario, A incurs a financial loss (Damnum) due to B's actions, but none of A's legal rights are violated. A's loss occurs because B is exercising his own 'legal right' to establish a business. Thus, there is damage to A, but it arises because B exercised his legal right, and none of A's legal rights were infringed upon. • This serves as an example of Damnum Sine Injuria, and such cases do not qualify as actionable torts.

Read about: How to Prepare for Judiciary Exams from Scratch

Therefore, for tortious liability to be established, there must be 'Injuria' - the violation of a legal right vested in an individual, while 'Damnum' - harm, loss, or damage, is irrelevant.

judiciary online coaching
judiciary online coaching

Distinguishing Civil Wrongs (Private Wrongs) from Crimes (Public Wrongs):

  1. Civil Wrongs, often referred to as Private wrongs, entail wrongful actions committed by one individual against another. These disputes exclusively involve private individuals, and the state does not intervene, as they do not pertain to matters of public interest.

    In contrast, Crimes are termed Public wrongs, as they encompass wrongful acts that impact not just specific individuals but society (the public) as a whole. These actions have repercussions that extend beyond individual grievances, affecting the larger community.

  1. Differentiating Civil Wrongs (Private Wrongs) from Crimes (Public Wrongs):

  2. In cases of civil wrongs, the primary remedy is Compensation (in cash or kind), as the objective is to restore the wronged person to their previous state.

    In contrast, criminal cases primarily seek Imprisonment as a remedy, with the aim of punishing the wrongdoer.

  3. In civil cases, both parties involved are typically private individuals. However, it is possible for the state to act as either the plaintiff or defendant, depending on the specific case's circumstances.

    In criminal cases, the State always assumes the role of the Prosecution and cannot be the Defendant.

  4. In civil cases, particularly in the context of torts, the wrongdoer is referred to as a Tort-feasor.

    In criminal cases, the wrongdoer is called an offender, criminal, or accused.

  5. Civil cases, including torts, are generally considered compoundable, allowing parties to reach a compromise outside of court, leading to the withdrawal of the case.

    Conversely, in most criminal cases involving public wrongs, compromises are not permitted, rendering them non-compoundable.

  6. Certain categories of wrongful acts can be classified as both Crimes and civil wrongs, with their categorization depending on the severity of the wrongdoing, determining whether they fall under the purview of Public wrongs or private wrongs. Examples of such acts include:

    i. Defamation. ii. Trespass iii. Assault iv. Negligence v. Nuisance

Know more: How to prepare for judiciary exams 

Download FREE Tort Notes for Judiciary Exam by Judiciary Gold

Types of Torts:

Tort law, being un-codified, cannot be confined to strict boundaries. Moreover, it constantly evolves through court adjudications. As previously discussed, tort law employs a negative definition, making it challenging to compile an exclusive list of wrongful acts classified as torts.

Any civil wrong that does not qualify as a Breach of Contract or Breach of Trust is considered a Tort. Therefore, there is no definitive list of acts that constitute torts.

However, some specific categories of torts are outlined below, while torts falling outside of these categories are considered miscellaneous and uncategorized:

Battery: Battery occurs when physical force is applied to another person in an offensive manner, resulting in harm.

Battery encompasses the deliberate application of force to another individual without legal justification.

  • Even if the party did not have the intention to harm the other person but was aware that their actions could potentially cause injury, a battery is still committed.
  • Striking someone in the face or using a bat to assault someone are typical examples of battery.

Get Details: Books for Judiciary Mains Exams 

Elements of Battery:

a) Use of Force: • Battery necessitates the application of force to establish the wrongful act. • Even if the force exerted is minimal, and no harm results, the wrongful act is still committed. • Physical injury is not a prerequisite; even the slightest touch in anger constitutes battery. • Examples of the use of force include slapping someone, wielding a stick to strike another person, throwing water, or spitting on someone.

b) Absence of Legal Justification: • It is crucial that the use of force is intentional and lacks any legal justification. • Harm resulting from the voluntary actions of the plaintiff does not fall under battery. • Harm that is unintentional or caused by pure accident is not actionable. For instance, if one person accidentally lightly touches another in a narrow tunnel, it does not constitute battery. However, if the touch is aggressive or excessive, it becomes actionable.

Elements of Assault: 3. Assault entails the reasonable apprehension of an impending battery upon another person. • The act of the defendant must generate a sense of impending battery in the plaintiff's mind for assault to occur. • Pointing a gun at someone, whether loaded or unloaded, creates a sense of impending harm in the mind of the individual targeted, thus constituting assault.

a) Intent to Cause Apprehension: • If the victim can demonstrate that there were reasonable circumstances leading them to believe that the tortfeasor (the person committing the tort) was about to cause harm, assault is established.

b) Imminent Harm: • There must be a foreseeable threat to the plaintiff, with a prima facie ability to execute the harm. For example, if a fist is displayed from such a distance that the threat cannot be carried out, there is no assault.

Comparison between Assault and Battery: ASSAULT BATTERY

  1. Assault involves creating an apprehension in a person's mind that a battery is about to be committed. Battery involves the actual use of force without lawful justification.
  2. Physical contact is not required. Physical contact is essential.
  3. The purpose of assault is to threaten. The purpose of battery is to cause harm.
  4. Example of assault: Pointing a gun at someone. Example of battery: Hitting someone with a stick.
  5. In most of the instances Assault (which is Mental) precedes Battery (which is physical).

Know More: Why solving previous year's question papers is important

False imprisonment

It refers to the unjust confinement of an individual, imposing a complete restriction on their freedom for a limited duration and without any lawful justification. • When a person's personal liberty is curtailed by confining them within four walls or by preventing them from leaving a specific location, it constitutes false imprisonment.

Essential Elements of False Imprisonment: a) Complete Restraint on an Individual's Liberty: • In criminal law, both total and partial restraint are actionable offenses. However, in tort, only total restraint is punishable. • If an individual is prevented from moving in a particular direction but is free to choose any other path, it does not amount to imprisonment. • To establish a false imprisonment offense, a person must be completely deprived of their liberty to move beyond a certain limit.

For Example: A section of a public footway was enclosed by the defendant, with seats placed there, and entry to the enclosure was restricted to those who paid to watch rowing. The plaintiff climbed over the enclosure's fence but was prevented from proceeding further. He filed a lawsuit for false imprisonment. The court ruled that there was no false imprisonment because there was no total restraint on the plaintiff's liberty. The plaintiff was free to return or even cross the bridge via the carriageway.

Read about: How to make a career in Judiciary

Availability of Means of Escape:

• If a means of escape exists, it does not qualify as false imprisonment. However, the means of escape should be intelligible to the detained person. For Example: If the detained person is visually impaired, they may not be able to locate the means of escape.

• The means of escape should also be reasonable. For Example: If the window providing an escape route is so high that it could cause injury to the detained person, it would still constitute false imprisonment.

Defamation:

• Defamation pertains to the injury inflicted upon a person's reputation, goodwill, and character. • An individual's reputation is considered their valuable property, often more precious than other possessions. Therefore, if someone tarnishes another person's reputation, they do so at their own risk.

Defamation comes in two forms:

i. Libel: This involves making a defamatory statement in a permanent form, such as in writing, printing, or pictures.

ii. Slander: This refers to the publication of a defamatory statement in a transient form, such as spoken words or gestures.

Essential Elements of Defamation:

a) The Statement Must Be Defamatory:

  • A defamatory statement is one that has the potential to harm the reputation of the plaintiff.
  • When a statement leads to feelings of hatred, contempt, ridicule, fear, dislike, or disesteem towards an individual, it is considered defamatory.

For instance, consider the following scenario: A local newspaper published a statement claiming that 'A' had eloped with 'C' the previous night. 'A' came from an educated family and was a final-year B.Com student. The published news item was entirely false, displayed extreme irresponsibility, and lacked any justification. This publication had detrimental effects on 'A's marriage prospects. The statement was deemed defamatory, and the defendants were held accountable.

b) The Statement Must Refer to the Plaintiff: • The plaintiff is required to demonstrate that the statement believed to be defamatory indeed pertains to the plaintiff. • It is irrelevant whether the defendant intended to defame the plaintiff or not.

c) The Statement Must Be Published: • Publication, in this context, means that the defamatory content must be made known to someone other than the person defamed, or else there is no basis for a civil action. • If a defamatory letter sent to the plaintiff in Urdu is read by a third party who is not authorized to do so, and it is proven that the defendant knew that the plaintiff did not understand Urdu when writing the letter, then the defendant can be held liable.

Trespass:

  • Trespass involves the unreasonable interference with property, land, a person, or goods.
  • In trespass, the owner's legal rights are violated because they are deprived of their right to enjoy the benefits of their property due to misappropriation or misuse of their rights.
  • Trespass can be categorized into three types:

i. Trespass to Person:

  • Trespass to person occurs when interference or harm is inflicted on an individual's body.
  • It constitutes an invasion of a person's freedom.
  • It encompasses the following torts: Assault, Battery, and False Imprisonment.
  • Examples include slapping someone or aiming a gun at someone.

ii. Trespass to Land:

  • Trespass to land involves interfering with the possession of land without lawful justification.
  • Trespass can occur when a person enters another person's land themselves or through a tangible object.
  • Exceeding the purpose for which a person entered certain premises or crossing a boundary where they have no authority to go constitutes trespass.
  • Trespass is not applicable if a person enters another's premises solely to save someone's life.

III. Trespass to Goods:

  • Trespass to goods involves interfering with goods that are in the plaintiff's possession, without any lawful justification.
  • Goods encompass personal property, whether movable or immovable, excluding land.
  • Mistaken ownership cannot be used as a defense. For instance, if two friends are studying together and after studying, B mistakenly takes A's book, thinking it's his own. In this case, the defense of mistaken ownership cannot be invoked.
  • If the defendant wrongfully holds goods that belong to the plaintiff and refuses to return them upon lawful demand, the plaintiff can recover the goods through a detinue action. ➢ If an individual intentionally and without any justification converts the property of another person who is entitled to immediate possession and use of the property, then that person is liable for the conversion of chattel.

Nuisance:

  • Nuisance entails the unlawful interference with a person's use or enjoyment of their land.
  • It includes actions that disrupt comfort, health, and safety. Interference can manifest in various forms, such as noise, vibrations, smoke, excavation, and more.
  • Nuisance differs from trespass in the following ways: i. In trespass, the interference is direct, whereas in nuisance, it is consequential. ii. Trespass always involves interference through tangible or physical objects, whereas nuisance can involve both tangible and intangible elements, such as smoke or vibrations. iii. Trespass is actionable per se (without the need for the plaintiff to prove actual damage), while in nuisance, special damage must be demonstrated.

Essential Elements of Nuisance:

a) Unreasonable Interference:

  • The interference may lead to damage to the plaintiff's property or cause personal discomfort in the plaintiff's enjoyment of the property.
  • Not every interference qualifies as a nuisance. For example, if someone owns a house beside a road, they cannot file a lawsuit for the inconveniences that are a necessary and incidental part of road traffic.
  • An unreasonable activity cannot be justified merely by demonstrating reasonable precautions taken to prevent it from becoming a nuisance.
  • An act that is reasonable does not become unreasonable and actionable solely due to the plaintiff's sensitiveness. For instance, if noise from a neighbor does not bother an ordinary person but only disturbs the plaintiff due to their heightened sensitivity, it does not constitute a nuisance.

b) Interference with the Enjoyment of Land:

The interference may cause either (a) harm to the property or (b) harm to comfort or health.

  • Harm to property: Unauthorized interference with another person's property usage, whether through tangible or intangible means, constitutes nuisance. An example is allowing tree branches to overhang onto another person's land.
  • Harm to comfort or health: Interference with comfort and convenience while using premises is actionable under nuisance, although the law does not concern itself with trivial matters.
  • [Point of Difference between Nuisance and Trespass: Nuisance may result from indirect physical interference, while trespass always involves direct physical interference by the defendant.]

Negligence:

  • Negligence refers to the absence of reasonable care that is expected from all individuals to prevent harm to others. • The burden of proof lies with the plaintiff to establish that the defendant was likely negligent.
  • For Example: Rohan, the owner of a large dog, leaves his dog unattended, and the dog attacks a passerby, causing severe injuries. In this case, Rohan's actions can be considered negligent.

Essential Elements of Negligence:

a) Duty of Care: The defendant must owe a duty of care to the plaintiff.

  1. The plaintiff must demonstrate that the defendant had a specific legal obligation to exercise care and that this duty was breached.
  2. Sometimes, the relationship between the plaintiff and defendant automatically imposes a legal duty or obligation to act in a certain manner. For instance, a doctor has a duty of care to provide appropriate treatment to a patient.

b) Breach of Duty: The defendant must have breached this duty. ➢ The defendant can be deemed negligent if they knew that their actions could harm another person but failed to act appropriately. For instance, if the defendant is aware that their dog is ferocious, they should place a warning sign outside their house, such as "Beware of Dog." If they fail to do so, they may be considered negligent.

c) Damage: The plaintiff must have suffered harm as a result of the defendant's actions. ➢ It is crucial to prove that the defendant's breach of duty directly caused harm to the plaintiff. ➢ The plaintiff must demonstrate that the damage is not too remote. ➢ The assessment of damage is at the court's discretion.

Nervous Shock:

  • Nervous shock occurs when a person experiences physical injury solely as a result of a shock to their nerves caused by something they have seen or heard.
  • Nervous shock is a psychiatric illness or injury resulting from events due to the negligence of another person.
  • It is a shock that arises from a reasonable fear of immediate personal harm.

For Example: • A falsely informs B's pregnant wife, C, that her husband, B, has suffered a nervous shock and has become seriously ill, breaking both legs in an accident. In this case, A was held liable for causing nervous shock.

General Defenses in Tort Law:

When the plaintiff (the party filing a lawsuit) brings an action against the defendant (the party being sued) for a specific tort, the defendant can avoid liability by asserting certain general defenses.

General Defenses vs. Specific Defenses:

  • General defenses in Tort law are defenses that can be used against various torts. For example, the defense of consent can be applied to cases of trespass, defamation, or other torts.
  • Specific defenses, on the other hand, are defenses unique to particular wrongs, such as the defenses of fair comment, privilege, or justification, which apply only to defamation cases.

General Defenses:

Volenti Non Fit Injuria (Consent):

  1. According to this maxim, "to a willing person, no injury is done." This means that a person who willingly and knowingly places themselves in a dangerous situation cannot sue for resulting injuries.
  2. When a person consents to the infliction of harm upon themselves, they can use the defense of Volenti Non Fit Injuria.
  3. Consent can be either express or implied.

Essential Conditions for Volenti Non Fit Injuria:

Free Consent: To assert this defense, it is crucial to demonstrate that the plaintiff's consent was given freely. If consent is obtained through fraud, compulsion, or under a mistaken impression, the defendant cannot rely on the Doctrine of Volenti Non Fit Injuria.

For Example: A married woman (plaintiff) noticed the development of a painful lump in her hand. The lump had no effect on her uterus, but during surgery, her uterus was removed without justification. It was held that the hospital (defendant) was liable for deficiency in service, and the plaintiff's consent for the hand operation did not imply consent for uterus removal.

  • Consent Cannot Be Given for an Unlawful Act: Consent for an unlawful act that is prohibited by law cannot legalize the act. For instance, consent to engage in a fight with sharp swords is unlawful, even if both parties consent.
  • Mere Knowledge Does Not Imply Assent: For the maxim Volenti Non Fit Injuria to apply, two points must be proven: (a) the plaintiff knew that there was a risk, and (b) knowing the risk, the plaintiff agreed to accept the harm. Both points must be established for this maxim to apply.

For Example:The plaintiff (a workman) was employed by the defendant for cutting rocks. There was a crane at the drilling site used to convey stones. The crane repeatedly passed over the plaintiff's head. While the plaintiff was working, a stone fell from the crane and injured him. The court held that there was only knowledge of the risk without the assumption of it. Therefore, the maxim Volenti Non Fit Injuria did not apply, and the defendants were held liable.

Plaintiff as the Wrongdoer:

  • When the plaintiff themselves engage in wrongful conduct that results in their injury, they cannot seek damages from the defendant.
  • This principle is rooted in the maxim "Ex turpi causa non oritur actio," which translates to "from an immoral cause, no action arises."

Inevitable Accident:

  • Inevitable accident occurs when an injury happens to a person due to an event that could not have been foreseen and prevented, even with reasonable care by the defendant.
  • To use this defense, two conditions must be met: (a) the defendant did not intend to harm the plaintiff, and (b) despite exercising reasonable care, the defendant could not have avoided the accident.

For Example: • The plaintiff and defendant went pheasant shooting. The defendant aimed at a pheasant, but the shot ricocheted off an oak tree, injuring the plaintiff. It was determined that the injury was accidental, an inevitable accident, and the defendant was not held liable.

For Example: • Two strangers accepted a ride in a jeep. Later, one of the bolts securing the right front wheel of the jeep to the axle failed, causing the wheel to detach, leading to a rollover accident resulting in injuries and one fatality. It was found to be a pure accident that could not have been detected through routine inspections. Therefore, the defendant was not held liable due to the defense of inevitable accident.

Act of God:

  • Act of God is a form of inevitable accident but differs in that it encompasses consequences arising from natural forces' operation, such as exceptionally heavy rainfall, volcanic eruptions, tides, etc.
  • Two essential conditions must be satisfied for this defense: The event causing damage must result from natural forces' operation without any human intervention. For example: • If the defendant's lorry carrying goods was looted by an unruly mob, it cannot be considered an Act of God, and the defendant is responsible for the lost goods.
  • The occurrence must be extraordinary and not something that could have been reasonably anticipated and prevented. For Example: A building's wall collapsed on a day with 2.66 inches of rainfall, resulting in the deaths of two children. The court ruled that the defendant could not use the Act of God defense since such rainfall during the rainy season is common and not extraordinary. Therefore, the defendant was held liable.

Know More: Why solving previous year's question papers is important

Private Defence:

When someone uses reasonable force to protect their personal safety or property, they may be excused if the force used was reasonable. For example, if someone is about to physically harm you, and in self-defense, you use force to protect yourself, your actions may be considered reasonable.

The essential requirements for this defense are as follows:

  1. The defendant must be facing an imminent threat to their personal safety or property.
  2. The defense must be in response to a genuine self-defense situation and not driven by a desire for revenge.
  3. The level of response must be proportionate to the nature of the attack or threat.

For Example: • Defendant A had placed live electric wire on his land. While crossing the land at 10 p.m. to reach his property, the plaintiff received a shock from the wire and suffered injuries. The defendant had not provided any visible warning about the wire. As a result, the defendant was held liable for the injuries caused to the plaintiff.

Mistake:

There are two types of mistakes in the context of tort law: a) Mistake of Fact b) Mistake of Law

Generally, both types of mistakes do not serve as defenses in tort law. However, there is an exception to this rule. If a defendant acts under an honest but mistaken belief, they may use the defense of Mistake to mitigate their liability under tort law.

For Example: • The defendant published a statement claiming that the plaintiff had given birth to twins in good faith. In reality, the plaintiff had only been married for two months. In this case, the defendant was held liable for defamation, and the element of good faith was considered immaterial.

Necessity:

An act carried out to prevent greater harm is not considered actionable, even if the harm was intentionally caused. In cases of necessity, it is crucial to demonstrate that the act was necessary to prevent a greater harm.

For instance, if someone throws goods off a ship to prevent the ship from sinking, this action falls under the defense of necessity.

It's important to note that necessity and private defense differ. In necessity, harm may be inflicted on an innocent party to prevent a greater harm, whereas in private defense, harm is caused to a plaintiff who is the wrongdoer.

Statutory Authority:

When damage results from an action that is authorized or mandated by a specific legislation or statute, it is not considered a basis for an actionable tort.

It serves as a complete defense, leaving the injured party with no recourse except for the damages specified in the statute. This immunity extends to all actions that are incidental to the exercise of such authority.

For Example: When activities such as noise, vibration, or smoke from running trains may potentially harm a person, no legal action can be taken except to seek compensation as provided for by the relevant statute.

For Example: The Respondent was authorized to operate railways. On a particular day, sparks from a train ignited a fire on the Plaintiff's adjacent lands. In this case, it was determined that the Defendant had taken reasonable precautions to prevent sparks from the train, and their actions were consistent with what the Statute had permitted them to do. Therefore, they were immune from liability and held not responsible.

Types of Torts:

  • Tort law, being uncodified, cannot be confined to specific boundaries. Furthermore, it is a branch of civil law that continually evolves through court decisions in various cases.
  • As previously discussed, the Law of Torts is defined negatively, making it difficult to provide an exclusive list of wrongful acts falling under the category of Torts.
  • Any civil wrongdoing that does not involve a Breach of Contract or Breach of Trust constitutes a Tort. Therefore, there is no definitive list of acts that qualify as torts.
  • However, there are specific categories of Torts, as outlined below, while other torts not falling into these categories are considered miscellaneous and uncategorized.

Read about: How to make a career in Judiciary

Download Important topics of Torts for Judiciary Exam

Battery:

Battery occurs when physical force is intentionally applied to another person in an offensive manner, resulting in some form of harm. It refers to the deliberate use of force on another person without legal justification. Even if the force used is minimal and causes no harm, a battery can still be committed.

Physical injury is not a prerequisite; even the slightest touch in anger constitutes a battery. Examples of the use of force include slapping a person, beating someone with a bat, or throwing water or spitting on someone.

Essential Elements of Battery:

a) Use of force:

  • The essential element of battery is the use of force.
  • Even if the force applied is insignificant and causes no harm, a battery can still be established.
  • Physical harm is not a requirement; the mere act of touching someone in anger qualifies as battery.

Examples of the use of force include slapping, hitting with an object, or spitting on someone.

b) Lack of lawful justification:

  • It is also crucial that the use of force is intentional and lacks lawful justification.
  • Harm resulting from the plaintiff's voluntary actions does not constitute battery.
  • Harm that is unintentional or the result of pure accident is not actionable. For instance, in a narrow tunnel, one person accidentally touches another gently, which does not amount to battery. However, if the touch is rude or excessive, it becomes actionable.

Assault:

  • Assault involves the creation of a reasonable apprehension in another person's mind that a battery is about to be committed.
  • Pointing a gun at someone, whether loaded or unloaded, creates an apprehension in the mind ofthe person on whom the gun is pointed. Hence the wrong of Assault has been committed.

Essential Elements of Assault Include:

a) Intent to cause apprehension:

  • The tortfeasor (the person committing the tort) must have the intention to create a reasonable apprehension in the victim's mind that harm is imminent.

b) Imminent harm:

  • There must be a clear and immediate threat to the plaintiff. The tortfeasor must have the apparent ability to carry out the harm. For instance, if a fist is shown from a distance that makes it impossible to execute the threat, then it does not constitute assault.

In most cases, Assault (which is mental) precedes Battery (which is physical).

c) False Imprisonment:

  • False imprisonment involves the imposition of complete restraint on an individual's liberty for a limited period and without any lawful justification.
  • It occurs when a person's freedom is unjustly taken away by confining them within four walls or by preventing them from leaving a specific location. Essential Elements of False Imprisonment Include: a) Total Restraint on an Individual's Liberty:
  • In criminal law, both total and partial restraint can be actionable offenses. However, in tort law, only total restraint is punishable.
  • False imprisonment occurs when a person is entirely deprived of their freedom to move beyond a specified limit, rather than simply being prevented from going in a particular direction while remaining free to choose another path.

Read moreShort tricks to write answers in Judiciary Mains Exam

b) Means of Escape:

  • False imprisonment may not apply if there is a viable means of escape. However, the means of escape should be comprehensible to the person who is detained. For instance, if the detained person is blind, they might not be able to locate the means of escape.
  • Additionally, the means of escape should be reasonable. If, for example, the window offering an escape route is too high and could cause injury to the detained person, then it may still be considered false imprisonment.

Defamation:

  • Defamation involves causing harm to an individual's reputation, goodwill, and character.
  • A person's reputation is regarded as a valuable asset, and anyone who injures another person's reputation does so at their own risk.
  • Defamation can be categorized into two types: i. Libel: This form of defamation occurs through representations made in a permanent form, such as in writing, printing, or pictures. ii. Slander: Slander pertains to the publication of defamatory statements in a transient form, such as spoken words or gestures.

Essential Elements of Defamation Include:

a) The Statement Must Be Defamatory:

A defamatory statement is one that tends to harm the reputation of the plaintiff.  If a statement causes someone to be regarded with feelings of hatred, contempt, ridicule, fear, dislike, or disesteem, it is considered defamatory. For example: A local newspaper published a false statement claiming that 'A' had eloped with 'C' the previous night. 'A' came from a well-educated family and was also a final-year BCom student. The news item was published irresponsibly and without any justification. This publication had a detrimental impact on 'A's' marriage prospects, and the statement was deemed defamatory, leading to liability for the defendants.

Trespass:

  • Trespass entails an unreasonable interference with property, land, a person, or goods.
  • In trespass, the legal rights of the owner are violated as they are deprived of their ability to enjoy the benefits of their property due to misappropriation or exploitation.
  • Trespass can be categorized into three types: i. Trespass to Person ii. Trespass to Land iii. Trespass to Goods

I. Trespass to Person: Occurs when there is interference or harm to a person's body, such as assault, battery, or false imprisonment.

  • It infringes upon a person's right to freedom.
  • It includes torts such as assault, battery, and false imprisonment. Example: Slapping someone or pointing a gun at them.

II. Trespass to Land: Involves interference with the possession of land without lawful justification.

  • Trespass can occur when a person enters another person's land themselves or through the use of some tangible object.
  • Going beyond the intended purpose for entering specific premises or crossing a boundary where one lacks authorization constitutes trespass.
  • If a person enters another's premises solely to save someone's life, it may not be considered trespass.

III. Trespass to Goods: Entails interference with goods in the plaintiff's possession without lawful justification.

  • Goods refer to personal property, whether movable or immovable, except for land.
  • Mistaken ownership cannot be used as a defense. For instance, if two friends are studying together and one mistakenly takes the other's book, this mistake of ownership cannot be a defense.
  • If the defendant wrongfully withholds goods belonging to the plaintiff and refuses to return them upon lawful demand, the plaintiff can recover them through an action for detinue.
  • If a person willfully and without any justification converts the goods of another person who is entitled to immediate use and possession of the goods, then the person is liable for conversion of chattel.

Read moreSimple tricks to enhance your English preparation for Judiciary Exams

Nuisance:

Nuisance refers to the unlawful interference with a person's use or enjoyment of land. Acts that interfere with comfort, health, safety, and so on are examples of nuisance. Interference can occur in various forms, such as noise, vibrations, smoke, excavation, and more.

Nuisance differs from trespass in several ways:

i. In trespass, interference is direct, whereas in nuisance, it is consequential.

ii. Trespass always involves physical or tangible objects, while nuisance can involve both tangible and intangible factors, like smoke or vibrations.

iii. Trespass is actionable per se, meaning the plaintiff doesn't need to prove actual damage, while in nuisance, special damage must be demonstrated.

Read About:  Judiciary Exam Preparation Tips

Essential elements of Nuisance:

a) Unreasonable interference:

  • Interference may result in damage to the plaintiff's property or personal discomfort in the enjoyment of the property.
  • Not all interference qualifies as a nuisance. For instance, if I own a house by the side of a road, I cannot bring a lawsuit for the inconvenience that is necessarily incidental to the traffic on the road.
  • An unreasonable activity cannot be justified simply by claiming that reasonable care was taken to prevent it from becoming a nuisance.
  • An act that is reasonable does not become unreasonable and actionable solely because the plaintiff is overly sensitive to it. For example, if noise from a neighbor doesn't bother an average person but disturbs the plaintiff due to their heightened sensitivity, it may not be considered a nuisance.

b) Interference with the enjoyment of land:

  • Interference may result in either (a) damage to property or (b) disruption of comfort or health.
  • Injury to property occurs when there is an unauthorized interference with the use of another person's property through tangible or intangible means, such as allowing tree branches to overhang onto another person's land.
  • Injury to comfort or health arises when there is interference with the comfort and convenience of using the premises. However, the law does not address trivial matters.

[Difference between Nuisance and Trespass – Nuisance may be caused by indirect physical interference, while Trespass always involves direct physical interference by the defendant.

For example: • If A throws a stone into B's house, it is considered Nuisance because the interference is indirect (by the stone), and A did not enter the premises. On the other hand, if A enters B's house without permission and without lawful justification, it would be considered Trespass because the interference is direct (resulting from the defendant's entry).]

Know more: How to prepare for judiciary exams 

Negligence:

  • Negligence refers to the absence of reasonable care that is expected from all individuals to prevent harm to others.
  • The burden of proof typically rests on the plaintiff to demonstrate that the defendant was likely negligent.

For example: Rohan, the owner of a large dog, left his dog unattended, and it attacked a passerby, causing severe injuries. In this case, it can be argued that Rohan's actions were negligent.

Essential elements of malicious prosecution:

a) Duty of care: The defendant must owe a duty of care to the plaintiff.

  • The plaintiff needs to establish that the defendant had a specific legal duty to exercise care and that this duty was breached.
  • Sometimes, the relationship between the plaintiff and defendant automatically gives rise to a legal duty or obligation to act in a certain manner. For instance, a doctor owes a duty of care to treat the patient appropriately.

b) Breach of duty: The defendant has breached the duty.

The defendant can be found negligent if they were aware that their actions could harm another person but did not act appropriately. For instance, if the defendant knows that their dog is aggressive, they should post a warning sign outside their house that says "Beware of dog." Failure to do so may be considered negligence.

c) Damage: The plaintiff must have suffered harm as a result of the defendant's actions. It is crucial to prove that the defendant's breach of duty directly caused harm to the plaintiff. The court has the discretion to assess the extent of the damage.

Nervous shock:

  • Nervous shock refers to when a person experiences physical injury as a result of a mental shock caused by something they have seen or heard.
  • Nervous shock is a psychological illness or injury that occurs due to the negligence of another person.
  • It occurs when there is a reasonable fear of immediate personal harm to oneself.

Get Details: Books for Judiciary Mains Exams 

For Example:

A person (A) falsely informed B's pregnant wife (C) that her husband (B) had suffered a nervous shock and had a serious accident resulting in both legs being broken. In this case, A was held responsible for causing nervous shock.

Questions from Law of Torts for Judiciary Exams 2023

Here are a few essential questions based on the law of torts for the Judicial Services Exam. Try answering these questions to check your preparation levels. Also, solving the previous year's Judiciary exam question papers will help understand the exam's difficulty level. 

Q1. Winfield defined 'tort' as

(a) a civil wrong for which the remedy is a common-law action for unliquidated damages and which is not exclusively the breach of trust or other merely eq­uitable obligation

(b) tortious liability arises from the breach of a duty primarily fixed by the law to­wards the persons generally, and its violation is redressible by an action for unliquidated damages

(c) an infringement of a right in rem of a private individual giving a right of compensation at the suit of the injured party

(d) none of the above.

Ans. Option b.

Q2. Tort is redressible by an action

(a) for restoration of the original position

(b) for unliquidated damages

(c) for liquidated damages

(d) all of the above.

Ans. Option b.

Q3. The 'tort of deceit' owes its origin to

(a) Pasley v. Freeman, (1789) 3 TR 51

(b) Lumley v. Gye, (1853) 2 E&B 216

(c) Rylands v. Fletcher, (1868) LR 3 HL 330

(d) Winsmore v. Greenbank, (1745) Willes 577.

Ans. Option a.

Q4. The 'tort of inducement a breach of contract' finds its origin in

(a) Lumley v. Gye

(b) Rookes v. Barnard

(c) Donoghue v. Stevenson

(d) Rylands v. Fletcher.

Ans. Option a.

Q5. The rule of 'strict liability is based on the decision in

(a) Donoghue v. Stevenson

(b) Rylands v. Fletcher

(c) Lumley v. Gye

(d) Champman v. Pickersgill.

Ans. Option b.

Q6. The tort is a violation of

(a) a right in personam

(b) a right in rem

(c) both right in personam & a right in rem

(d) neither a right in personam nor a right in rem.

Ans. Option b

Q7. Maxim injuria sine damno means

(a)violation of a legal right without any damage

(b) violation of a lawful request with damage

(c) damage without violation of the legal right

(d) no damage & no breach of legal rights.

Ans. Option a.

Learn more: Judiciary Exam 2023 Online Coaching

Q8. Malice in law means

(a) a wrongful act was done intentionally but without just cause or excuse

(b) a wrongful act was done intentionally with just cause & excuse

(c) a wrongful act was done intentionally with a good motive

(d) a wrongful act was done intentionally with an evil motive.

Ans. Option a.

Q9. The defence of volenti non-fit injuria, is not available

(a) if the consent is obtained by compul­sion

(b) if the consent is obtained by fraud

(c) if the consent is obtained under a mis­take

(d) all of the above.

Ans. Option d.

Q10. Inevitable accident means

(a) an act of God

(b) an unexpected injury which could have been foreseen & avoided

(c) unforeseen damage which could not have been expected & avoided

(d) both (a) & (b).

Ans. Option c.

Q11. Act of State

(a) cannot be between a sovereign and the subjects of another State

(b) cannot be between a ruler and his subjects

(c) both (a) & (b)

(d) neither (a) nor (b).

Ans. Option a.

Q12. The liability of joint tortfeasors is

(a) only joint

(b) only several

(c) joint & several

(d) neither joint nor several.

Ans. Option c.

Download free: Judiciary Exams Question Paper 

Q13. In the case of independent tort features

(a) action against one bar the action against the others

(b) action against some bars the action against the others

(c) action against one does not bar action against the other

(d) both (a) & (b).

Ans. Option b.

Q14. Vicarious liability commensurates with that of

(a) independent tort feasors

(b) joint tort feasors

(c) composite tort feasors

(d) both (a) & (c).

Ans. Option b.

Q15. For vicarious liability, a ser­vant is a person

(a) on whom the master has command of what to do & how to do

(b) on whom the master has a command of what to do

(c) whom the master can hire & fire

(d) only (a) & (c) above.

Ans. Option d.

Q16. Vicarious liability of master arises

(a) in case of theft of property by a third person, bailed to the master

(b) in case of theft of property of a third person not bailed to the master

(c) neither (a) nor (b) as the act of commit­ting theft is not covered

(d) Both (a) and (b) as it makes no differ­ence whether the goods are bailed or not bailed to the master.

Ans. Option b.

Q17. The doctrine of common employment means

(a) if the wrongdoer servant and the per­-son injured are fellow servants, the master is not liable

(b) if the wrongdoer servant and the per-­son injured are fellow servants, the master is liable

(c) if the wrongdoer servant and the per-­son injured are not fellow servants, the master is not liable

(d) none of the above.

Ans. Option a.

Also Read: Judiciary Interview Preparation Strategies & Tricks

Q18. In India, the scope of the doctrine of com­mon employment was restricted by

(a) The Indian Employers' Liability Act, 1938

(b) The Workmen's Compensation Act, 1923

(c) The Employees' State Insurance Act, 1948

(d) all of the above.

Ans. Option a.

Q19. ï»¿Remoteness of damages is determined by

(a) the test of directness

(b) the test of reasonable foresight

(c) both (a) & (b)

(d) none of the above.

Ans. Option b.

Q20. The test of reasonable foresight in deter­mining the remoteness of damages was first applied in

(a) Re: Polerris

(b) Wagon Mannd case

(c) Doughty v. Turner Manufacturing Co. Ltd.

(d) SCM (United Kingdom) Ltd. v. W.J. Whittal & Sons.

Ans. Option b.

Q21. For 'false imprisonment', there should be

(a) total restraint on the liberty of a person

(b) a partial restraint on the liberty of a person

(c) means of escape

(d) all of the above.

Ans. Option a.

Ace the exam with the best PCS J online coaching classes.

Conclusion:

For Torts an aspirtant must start their preparation for torts as soon as you decide that you are preparing for a state that covers torts in their syllabus. Though the concepts seem easy in torts but it is important to start its preapration from the starting only and keep revising it on regular basis.

All the best Aspirants!

Frequently Asked Questions

What are the stages in the Delhi Judiciary Exam Selection Process?

Expand Faq Icon

Which are important topics in Judiciary Prelims Exam?

Expand Faq Icon

What is the marking scheme of Judiciary Prelims and Mains Exam?

Expand Faq Icon

Is Viva-Voce stage compulsory?

Expand Faq Icon

What is the marking scheme of Judiciary Viva-Voce stage?

Expand Faq Icon

Do really solving Judiciary prelims Question Papers will help score better in the exam?

Expand Faq Icon

How to download Delhi judiciary Civil Law previous year question papers?

Expand Faq Icon

How to download Haryana judiciary General Knowledge previous year question papers?

Expand Faq Icon

Laws of Torts Notes [ Download PDF Free ]

Author : Yogricha

June 25, 2024

SHARE

Overview: The Law of Torts originated within the realm of common law, while Breach of Trust cases were typically addressed in the Chancery courts. Furthermore, the Law of Torts is not considered a subset of property law, whereas the Law of Trusts can be and is classified as a branch of property law.

In judiciary examinations like Uttar Pradesh Judiciary and other Judiciary Exams tort is an important subject that is asked in Prelims or Mains or both the satges of examination. If the state you are targetting has Law of Torts  In this Blog we will cover:

  • Imprortant topics of Tort
  • Practice Questions for Tort
  • Free notes for Judiciary preparation

Download FREE Study Material for Judiciary Exam by Judiciary Gold

Introduction to Law of Torts

The term 'Torts' finds its etymological roots in the Latin word "tortrum," signifying "to twist," while its French counterpart directly translates to a 'wrong.' In essence, a tort is a civil wrong, a private transgression that does not fall under the categories of Breach of Contract or Breach of Trust. To put it simply, any civil wrongdoing that cannot be classified as either a 'Breach of Contract' or a 'Breach of Trust' constitutes a tort.

Attempting to confine the concept of torts within a single, satisfactory definition is akin to trying to confine it within a rigid framework from which it perpetually seeks to break free.

Read about: Upcoming Judiciary Exams

A wrongful act can take the form of either a Public Wrong (a crime) or a Private Wrong (a civil wrong). Civil wrongs are further subdivided into three categories:

i. Breach of Contract ii. Breach of Trust iii. Tort

To establish whether a civil wrong qualifies as a tort, it must first be determined that the wrongdoing does not fall within the purview of the first two categories—Breach of Contract or Breach of Trust. If it does not fit into either of these categories, it is categorized as a tort. Consequently, the determination of whether an act constitutes a tort relies on a negative definition, employing a method of elimination.

The person committing a tort is referred to as a 'Tort-feasor.'

Tort stands as the only branch of law that remains un-codified, drawing its origins from the rulings of the House of Lords in England. The development and creation of tort law are shaped by court decisions, as it is not enshrined within a specific legislative act or statute.

Take a class: POCSO Act for Judiciary

A tort is committed whenever someone's act or omission violates a legal right vested in another person. This branch of law holds great significance in common law countries like India.

Essential Elements of Tort: To constitute an actionable tort, certain essential elements or prerequisites must be present, including:

I. Legal Injury (Injuria) – Legal injury signifies the violation of a legal right held by an individual.

II. Wrongful Act or Omission – The infliction of legal injury must stem from an individual's wrongful action or failure to act (omission). A wrongful act or omission arises when a person has a duty of care and breaches that duty, leading to legal injury to another party.

Download Free Notes: POCSO for Judiciary Examination

III. Availability of Legal Remedy – In order for a tort to be considered actionable, it is imperative that the wrongful act or omission falls within a category that permits the initiation of a civil lawsuit seeking damages (compensation).

The Two Fundamental Principles (Maxims) of Tort Law:

Tort law rests upon two fundamental maxims that are crucial for comprehending the essence of torts and how a tort becomes actionable:

  1. Injuria Sine Damnum: This maxim literally translates to 'Legal Injury (Injuria) without (Sine) Damage/Loss (Damnum).' In essence, Injuria Sine Damnum encompasses situations in which a person's legal rights are violated, yet no actual loss or damage is suffered as a consequence. Cases falling under 'Injuria Sine Damnum' constitute actionable torts. Therefore, the individual who experiences a legal injury is entitled to seek damages (compensation) from the court.

    Example: • A, a 21-year-old Indian citizen, was forcibly prevented by B from casting his vote in the General elections of his constituency. However, when the election results were announced, the candidate whom A intended to vote for emerged as the winner. • In this scenario, A's 'right to vote' (a legal right) was infringed upon by B's actions, but A did not endure any tangible loss since his preferred candidate won. Consequently, there is a Legal Injury (Injuria) without any accompanying damage or loss (Damnum). • This is a classic illustration of Injuria Sine Damnum, making it an actionable tort.

  2. Damnum Sine Injuria: This maxim literally means 'Damage (Damnum) without (Sine) a legal injury (Injuria).' Cases falling under this category do not qualify as actionable torts, and the plaintiff is not entitled to damages (compensation) even if they have incurred losses. This is because the key element required for tortious liability to arise is the presence of 'Legal Injury,' which is absent in such cases.

    Example: • A owns a bakery shop, and B is his employee. A enjoys a complete monopoly in the market. After some time, B opens a shop in A's vicinity, leading to competition in the market and a division of customers, resulting in significant financial losses for A. • In this scenario, A incurs a financial loss (Damnum) due to B's actions, but none of A's legal rights are violated. A's loss occurs because B is exercising his own 'legal right' to establish a business. Thus, there is damage to A, but it arises because B exercised his legal right, and none of A's legal rights were infringed upon. • This serves as an example of Damnum Sine Injuria, and such cases do not qualify as actionable torts.

Read about: How to Prepare for Judiciary Exams from Scratch

Therefore, for tortious liability to be established, there must be 'Injuria' - the violation of a legal right vested in an individual, while 'Damnum' - harm, loss, or damage, is irrelevant.

judiciary online coaching
judiciary online coaching

Distinguishing Civil Wrongs (Private Wrongs) from Crimes (Public Wrongs):

  1. Civil Wrongs, often referred to as Private wrongs, entail wrongful actions committed by one individual against another. These disputes exclusively involve private individuals, and the state does not intervene, as they do not pertain to matters of public interest.

    In contrast, Crimes are termed Public wrongs, as they encompass wrongful acts that impact not just specific individuals but society (the public) as a whole. These actions have repercussions that extend beyond individual grievances, affecting the larger community.

  1. Differentiating Civil Wrongs (Private Wrongs) from Crimes (Public Wrongs):

  2. In cases of civil wrongs, the primary remedy is Compensation (in cash or kind), as the objective is to restore the wronged person to their previous state.

    In contrast, criminal cases primarily seek Imprisonment as a remedy, with the aim of punishing the wrongdoer.

  3. In civil cases, both parties involved are typically private individuals. However, it is possible for the state to act as either the plaintiff or defendant, depending on the specific case's circumstances.

    In criminal cases, the State always assumes the role of the Prosecution and cannot be the Defendant.

  4. In civil cases, particularly in the context of torts, the wrongdoer is referred to as a Tort-feasor.

    In criminal cases, the wrongdoer is called an offender, criminal, or accused.

  5. Civil cases, including torts, are generally considered compoundable, allowing parties to reach a compromise outside of court, leading to the withdrawal of the case.

    Conversely, in most criminal cases involving public wrongs, compromises are not permitted, rendering them non-compoundable.

  6. Certain categories of wrongful acts can be classified as both Crimes and civil wrongs, with their categorization depending on the severity of the wrongdoing, determining whether they fall under the purview of Public wrongs or private wrongs. Examples of such acts include:

    i. Defamation. ii. Trespass iii. Assault iv. Negligence v. Nuisance

Know more: How to prepare for judiciary exams 

Download FREE Tort Notes for Judiciary Exam by Judiciary Gold

Types of Torts:

Tort law, being un-codified, cannot be confined to strict boundaries. Moreover, it constantly evolves through court adjudications. As previously discussed, tort law employs a negative definition, making it challenging to compile an exclusive list of wrongful acts classified as torts.

Any civil wrong that does not qualify as a Breach of Contract or Breach of Trust is considered a Tort. Therefore, there is no definitive list of acts that constitute torts.

However, some specific categories of torts are outlined below, while torts falling outside of these categories are considered miscellaneous and uncategorized:

Battery: Battery occurs when physical force is applied to another person in an offensive manner, resulting in harm.

Battery encompasses the deliberate application of force to another individual without legal justification.

  • Even if the party did not have the intention to harm the other person but was aware that their actions could potentially cause injury, a battery is still committed.
  • Striking someone in the face or using a bat to assault someone are typical examples of battery.

Get Details: Books for Judiciary Mains Exams 

Elements of Battery:

a) Use of Force: • Battery necessitates the application of force to establish the wrongful act. • Even if the force exerted is minimal, and no harm results, the wrongful act is still committed. • Physical injury is not a prerequisite; even the slightest touch in anger constitutes battery. • Examples of the use of force include slapping someone, wielding a stick to strike another person, throwing water, or spitting on someone.

b) Absence of Legal Justification: • It is crucial that the use of force is intentional and lacks any legal justification. • Harm resulting from the voluntary actions of the plaintiff does not fall under battery. • Harm that is unintentional or caused by pure accident is not actionable. For instance, if one person accidentally lightly touches another in a narrow tunnel, it does not constitute battery. However, if the touch is aggressive or excessive, it becomes actionable.

Elements of Assault: 3. Assault entails the reasonable apprehension of an impending battery upon another person. • The act of the defendant must generate a sense of impending battery in the plaintiff's mind for assault to occur. • Pointing a gun at someone, whether loaded or unloaded, creates a sense of impending harm in the mind of the individual targeted, thus constituting assault.

a) Intent to Cause Apprehension: • If the victim can demonstrate that there were reasonable circumstances leading them to believe that the tortfeasor (the person committing the tort) was about to cause harm, assault is established.

b) Imminent Harm: • There must be a foreseeable threat to the plaintiff, with a prima facie ability to execute the harm. For example, if a fist is displayed from such a distance that the threat cannot be carried out, there is no assault.

Comparison between Assault and Battery: ASSAULT BATTERY

  1. Assault involves creating an apprehension in a person's mind that a battery is about to be committed. Battery involves the actual use of force without lawful justification.
  2. Physical contact is not required. Physical contact is essential.
  3. The purpose of assault is to threaten. The purpose of battery is to cause harm.
  4. Example of assault: Pointing a gun at someone. Example of battery: Hitting someone with a stick.
  5. In most of the instances Assault (which is Mental) precedes Battery (which is physical).

Know More: Why solving previous year's question papers is important

False imprisonment

It refers to the unjust confinement of an individual, imposing a complete restriction on their freedom for a limited duration and without any lawful justification. • When a person's personal liberty is curtailed by confining them within four walls or by preventing them from leaving a specific location, it constitutes false imprisonment.

Essential Elements of False Imprisonment: a) Complete Restraint on an Individual's Liberty: • In criminal law, both total and partial restraint are actionable offenses. However, in tort, only total restraint is punishable. • If an individual is prevented from moving in a particular direction but is free to choose any other path, it does not amount to imprisonment. • To establish a false imprisonment offense, a person must be completely deprived of their liberty to move beyond a certain limit.

For Example: A section of a public footway was enclosed by the defendant, with seats placed there, and entry to the enclosure was restricted to those who paid to watch rowing. The plaintiff climbed over the enclosure's fence but was prevented from proceeding further. He filed a lawsuit for false imprisonment. The court ruled that there was no false imprisonment because there was no total restraint on the plaintiff's liberty. The plaintiff was free to return or even cross the bridge via the carriageway.

Read about: How to make a career in Judiciary

Availability of Means of Escape:

• If a means of escape exists, it does not qualify as false imprisonment. However, the means of escape should be intelligible to the detained person. For Example: If the detained person is visually impaired, they may not be able to locate the means of escape.

• The means of escape should also be reasonable. For Example: If the window providing an escape route is so high that it could cause injury to the detained person, it would still constitute false imprisonment.

Defamation:

• Defamation pertains to the injury inflicted upon a person's reputation, goodwill, and character. • An individual's reputation is considered their valuable property, often more precious than other possessions. Therefore, if someone tarnishes another person's reputation, they do so at their own risk.

Defamation comes in two forms:

i. Libel: This involves making a defamatory statement in a permanent form, such as in writing, printing, or pictures.

ii. Slander: This refers to the publication of a defamatory statement in a transient form, such as spoken words or gestures.

Essential Elements of Defamation:

a) The Statement Must Be Defamatory:

  • A defamatory statement is one that has the potential to harm the reputation of the plaintiff.
  • When a statement leads to feelings of hatred, contempt, ridicule, fear, dislike, or disesteem towards an individual, it is considered defamatory.

For instance, consider the following scenario: A local newspaper published a statement claiming that 'A' had eloped with 'C' the previous night. 'A' came from an educated family and was a final-year B.Com student. The published news item was entirely false, displayed extreme irresponsibility, and lacked any justification. This publication had detrimental effects on 'A's marriage prospects. The statement was deemed defamatory, and the defendants were held accountable.

b) The Statement Must Refer to the Plaintiff: • The plaintiff is required to demonstrate that the statement believed to be defamatory indeed pertains to the plaintiff. • It is irrelevant whether the defendant intended to defame the plaintiff or not.

c) The Statement Must Be Published: • Publication, in this context, means that the defamatory content must be made known to someone other than the person defamed, or else there is no basis for a civil action. • If a defamatory letter sent to the plaintiff in Urdu is read by a third party who is not authorized to do so, and it is proven that the defendant knew that the plaintiff did not understand Urdu when writing the letter, then the defendant can be held liable.

Trespass:

  • Trespass involves the unreasonable interference with property, land, a person, or goods.
  • In trespass, the owner's legal rights are violated because they are deprived of their right to enjoy the benefits of their property due to misappropriation or misuse of their rights.
  • Trespass can be categorized into three types:

i. Trespass to Person:

  • Trespass to person occurs when interference or harm is inflicted on an individual's body.
  • It constitutes an invasion of a person's freedom.
  • It encompasses the following torts: Assault, Battery, and False Imprisonment.
  • Examples include slapping someone or aiming a gun at someone.

ii. Trespass to Land:

  • Trespass to land involves interfering with the possession of land without lawful justification.
  • Trespass can occur when a person enters another person's land themselves or through a tangible object.
  • Exceeding the purpose for which a person entered certain premises or crossing a boundary where they have no authority to go constitutes trespass.
  • Trespass is not applicable if a person enters another's premises solely to save someone's life.

III. Trespass to Goods:

  • Trespass to goods involves interfering with goods that are in the plaintiff's possession, without any lawful justification.
  • Goods encompass personal property, whether movable or immovable, excluding land.
  • Mistaken ownership cannot be used as a defense. For instance, if two friends are studying together and after studying, B mistakenly takes A's book, thinking it's his own. In this case, the defense of mistaken ownership cannot be invoked.
  • If the defendant wrongfully holds goods that belong to the plaintiff and refuses to return them upon lawful demand, the plaintiff can recover the goods through a detinue action. ➢ If an individual intentionally and without any justification converts the property of another person who is entitled to immediate possession and use of the property, then that person is liable for the conversion of chattel.

Nuisance:

  • Nuisance entails the unlawful interference with a person's use or enjoyment of their land.
  • It includes actions that disrupt comfort, health, and safety. Interference can manifest in various forms, such as noise, vibrations, smoke, excavation, and more.
  • Nuisance differs from trespass in the following ways: i. In trespass, the interference is direct, whereas in nuisance, it is consequential. ii. Trespass always involves interference through tangible or physical objects, whereas nuisance can involve both tangible and intangible elements, such as smoke or vibrations. iii. Trespass is actionable per se (without the need for the plaintiff to prove actual damage), while in nuisance, special damage must be demonstrated.

Essential Elements of Nuisance:

a) Unreasonable Interference:

  • The interference may lead to damage to the plaintiff's property or cause personal discomfort in the plaintiff's enjoyment of the property.
  • Not every interference qualifies as a nuisance. For example, if someone owns a house beside a road, they cannot file a lawsuit for the inconveniences that are a necessary and incidental part of road traffic.
  • An unreasonable activity cannot be justified merely by demonstrating reasonable precautions taken to prevent it from becoming a nuisance.
  • An act that is reasonable does not become unreasonable and actionable solely due to the plaintiff's sensitiveness. For instance, if noise from a neighbor does not bother an ordinary person but only disturbs the plaintiff due to their heightened sensitivity, it does not constitute a nuisance.

b) Interference with the Enjoyment of Land:

The interference may cause either (a) harm to the property or (b) harm to comfort or health.

  • Harm to property: Unauthorized interference with another person's property usage, whether through tangible or intangible means, constitutes nuisance. An example is allowing tree branches to overhang onto another person's land.
  • Harm to comfort or health: Interference with comfort and convenience while using premises is actionable under nuisance, although the law does not concern itself with trivial matters.
  • [Point of Difference between Nuisance and Trespass: Nuisance may result from indirect physical interference, while trespass always involves direct physical interference by the defendant.]

Negligence:

  • Negligence refers to the absence of reasonable care that is expected from all individuals to prevent harm to others. • The burden of proof lies with the plaintiff to establish that the defendant was likely negligent.
  • For Example: Rohan, the owner of a large dog, leaves his dog unattended, and the dog attacks a passerby, causing severe injuries. In this case, Rohan's actions can be considered negligent.

Essential Elements of Negligence:

a) Duty of Care: The defendant must owe a duty of care to the plaintiff.

  1. The plaintiff must demonstrate that the defendant had a specific legal obligation to exercise care and that this duty was breached.
  2. Sometimes, the relationship between the plaintiff and defendant automatically imposes a legal duty or obligation to act in a certain manner. For instance, a doctor has a duty of care to provide appropriate treatment to a patient.

b) Breach of Duty: The defendant must have breached this duty. ➢ The defendant can be deemed negligent if they knew that their actions could harm another person but failed to act appropriately. For instance, if the defendant is aware that their dog is ferocious, they should place a warning sign outside their house, such as "Beware of Dog." If they fail to do so, they may be considered negligent.

c) Damage: The plaintiff must have suffered harm as a result of the defendant's actions. ➢ It is crucial to prove that the defendant's breach of duty directly caused harm to the plaintiff. ➢ The plaintiff must demonstrate that the damage is not too remote. ➢ The assessment of damage is at the court's discretion.

Nervous Shock:

  • Nervous shock occurs when a person experiences physical injury solely as a result of a shock to their nerves caused by something they have seen or heard.
  • Nervous shock is a psychiatric illness or injury resulting from events due to the negligence of another person.
  • It is a shock that arises from a reasonable fear of immediate personal harm.

For Example: • A falsely informs B's pregnant wife, C, that her husband, B, has suffered a nervous shock and has become seriously ill, breaking both legs in an accident. In this case, A was held liable for causing nervous shock.

General Defenses in Tort Law:

When the plaintiff (the party filing a lawsuit) brings an action against the defendant (the party being sued) for a specific tort, the defendant can avoid liability by asserting certain general defenses.

General Defenses vs. Specific Defenses:

  • General defenses in Tort law are defenses that can be used against various torts. For example, the defense of consent can be applied to cases of trespass, defamation, or other torts.
  • Specific defenses, on the other hand, are defenses unique to particular wrongs, such as the defenses of fair comment, privilege, or justification, which apply only to defamation cases.

General Defenses:

Volenti Non Fit Injuria (Consent):

  1. According to this maxim, "to a willing person, no injury is done." This means that a person who willingly and knowingly places themselves in a dangerous situation cannot sue for resulting injuries.
  2. When a person consents to the infliction of harm upon themselves, they can use the defense of Volenti Non Fit Injuria.
  3. Consent can be either express or implied.

Essential Conditions for Volenti Non Fit Injuria:

Free Consent: To assert this defense, it is crucial to demonstrate that the plaintiff's consent was given freely. If consent is obtained through fraud, compulsion, or under a mistaken impression, the defendant cannot rely on the Doctrine of Volenti Non Fit Injuria.

For Example: A married woman (plaintiff) noticed the development of a painful lump in her hand. The lump had no effect on her uterus, but during surgery, her uterus was removed without justification. It was held that the hospital (defendant) was liable for deficiency in service, and the plaintiff's consent for the hand operation did not imply consent for uterus removal.

  • Consent Cannot Be Given for an Unlawful Act: Consent for an unlawful act that is prohibited by law cannot legalize the act. For instance, consent to engage in a fight with sharp swords is unlawful, even if both parties consent.
  • Mere Knowledge Does Not Imply Assent: For the maxim Volenti Non Fit Injuria to apply, two points must be proven: (a) the plaintiff knew that there was a risk, and (b) knowing the risk, the plaintiff agreed to accept the harm. Both points must be established for this maxim to apply.

For Example:The plaintiff (a workman) was employed by the defendant for cutting rocks. There was a crane at the drilling site used to convey stones. The crane repeatedly passed over the plaintiff's head. While the plaintiff was working, a stone fell from the crane and injured him. The court held that there was only knowledge of the risk without the assumption of it. Therefore, the maxim Volenti Non Fit Injuria did not apply, and the defendants were held liable.

Plaintiff as the Wrongdoer:

  • When the plaintiff themselves engage in wrongful conduct that results in their injury, they cannot seek damages from the defendant.
  • This principle is rooted in the maxim "Ex turpi causa non oritur actio," which translates to "from an immoral cause, no action arises."

Inevitable Accident:

  • Inevitable accident occurs when an injury happens to a person due to an event that could not have been foreseen and prevented, even with reasonable care by the defendant.
  • To use this defense, two conditions must be met: (a) the defendant did not intend to harm the plaintiff, and (b) despite exercising reasonable care, the defendant could not have avoided the accident.

For Example: • The plaintiff and defendant went pheasant shooting. The defendant aimed at a pheasant, but the shot ricocheted off an oak tree, injuring the plaintiff. It was determined that the injury was accidental, an inevitable accident, and the defendant was not held liable.

For Example: • Two strangers accepted a ride in a jeep. Later, one of the bolts securing the right front wheel of the jeep to the axle failed, causing the wheel to detach, leading to a rollover accident resulting in injuries and one fatality. It was found to be a pure accident that could not have been detected through routine inspections. Therefore, the defendant was not held liable due to the defense of inevitable accident.

Act of God:

  • Act of God is a form of inevitable accident but differs in that it encompasses consequences arising from natural forces' operation, such as exceptionally heavy rainfall, volcanic eruptions, tides, etc.
  • Two essential conditions must be satisfied for this defense: The event causing damage must result from natural forces' operation without any human intervention. For example: • If the defendant's lorry carrying goods was looted by an unruly mob, it cannot be considered an Act of God, and the defendant is responsible for the lost goods.
  • The occurrence must be extraordinary and not something that could have been reasonably anticipated and prevented. For Example: A building's wall collapsed on a day with 2.66 inches of rainfall, resulting in the deaths of two children. The court ruled that the defendant could not use the Act of God defense since such rainfall during the rainy season is common and not extraordinary. Therefore, the defendant was held liable.

Know More: Why solving previous year's question papers is important

Private Defence:

When someone uses reasonable force to protect their personal safety or property, they may be excused if the force used was reasonable. For example, if someone is about to physically harm you, and in self-defense, you use force to protect yourself, your actions may be considered reasonable.

The essential requirements for this defense are as follows:

  1. The defendant must be facing an imminent threat to their personal safety or property.
  2. The defense must be in response to a genuine self-defense situation and not driven by a desire for revenge.
  3. The level of response must be proportionate to the nature of the attack or threat.

For Example: • Defendant A had placed live electric wire on his land. While crossing the land at 10 p.m. to reach his property, the plaintiff received a shock from the wire and suffered injuries. The defendant had not provided any visible warning about the wire. As a result, the defendant was held liable for the injuries caused to the plaintiff.

Mistake:

There are two types of mistakes in the context of tort law: a) Mistake of Fact b) Mistake of Law

Generally, both types of mistakes do not serve as defenses in tort law. However, there is an exception to this rule. If a defendant acts under an honest but mistaken belief, they may use the defense of Mistake to mitigate their liability under tort law.

For Example: • The defendant published a statement claiming that the plaintiff had given birth to twins in good faith. In reality, the plaintiff had only been married for two months. In this case, the defendant was held liable for defamation, and the element of good faith was considered immaterial.

Necessity:

An act carried out to prevent greater harm is not considered actionable, even if the harm was intentionally caused. In cases of necessity, it is crucial to demonstrate that the act was necessary to prevent a greater harm.

For instance, if someone throws goods off a ship to prevent the ship from sinking, this action falls under the defense of necessity.

It's important to note that necessity and private defense differ. In necessity, harm may be inflicted on an innocent party to prevent a greater harm, whereas in private defense, harm is caused to a plaintiff who is the wrongdoer.

Statutory Authority:

When damage results from an action that is authorized or mandated by a specific legislation or statute, it is not considered a basis for an actionable tort.

It serves as a complete defense, leaving the injured party with no recourse except for the damages specified in the statute. This immunity extends to all actions that are incidental to the exercise of such authority.

For Example: When activities such as noise, vibration, or smoke from running trains may potentially harm a person, no legal action can be taken except to seek compensation as provided for by the relevant statute.

For Example: The Respondent was authorized to operate railways. On a particular day, sparks from a train ignited a fire on the Plaintiff's adjacent lands. In this case, it was determined that the Defendant had taken reasonable precautions to prevent sparks from the train, and their actions were consistent with what the Statute had permitted them to do. Therefore, they were immune from liability and held not responsible.

Types of Torts:

  • Tort law, being uncodified, cannot be confined to specific boundaries. Furthermore, it is a branch of civil law that continually evolves through court decisions in various cases.
  • As previously discussed, the Law of Torts is defined negatively, making it difficult to provide an exclusive list of wrongful acts falling under the category of Torts.
  • Any civil wrongdoing that does not involve a Breach of Contract or Breach of Trust constitutes a Tort. Therefore, there is no definitive list of acts that qualify as torts.
  • However, there are specific categories of Torts, as outlined below, while other torts not falling into these categories are considered miscellaneous and uncategorized.

Read about: How to make a career in Judiciary

Download Important topics of Torts for Judiciary Exam

Battery:

Battery occurs when physical force is intentionally applied to another person in an offensive manner, resulting in some form of harm. It refers to the deliberate use of force on another person without legal justification. Even if the force used is minimal and causes no harm, a battery can still be committed.

Physical injury is not a prerequisite; even the slightest touch in anger constitutes a battery. Examples of the use of force include slapping a person, beating someone with a bat, or throwing water or spitting on someone.

Essential Elements of Battery:

a) Use of force:

  • The essential element of battery is the use of force.
  • Even if the force applied is insignificant and causes no harm, a battery can still be established.
  • Physical harm is not a requirement; the mere act of touching someone in anger qualifies as battery.

Examples of the use of force include slapping, hitting with an object, or spitting on someone.

b) Lack of lawful justification:

  • It is also crucial that the use of force is intentional and lacks lawful justification.
  • Harm resulting from the plaintiff's voluntary actions does not constitute battery.
  • Harm that is unintentional or the result of pure accident is not actionable. For instance, in a narrow tunnel, one person accidentally touches another gently, which does not amount to battery. However, if the touch is rude or excessive, it becomes actionable.

Assault:

  • Assault involves the creation of a reasonable apprehension in another person's mind that a battery is about to be committed.
  • Pointing a gun at someone, whether loaded or unloaded, creates an apprehension in the mind ofthe person on whom the gun is pointed. Hence the wrong of Assault has been committed.

Essential Elements of Assault Include:

a) Intent to cause apprehension:

  • The tortfeasor (the person committing the tort) must have the intention to create a reasonable apprehension in the victim's mind that harm is imminent.

b) Imminent harm:

  • There must be a clear and immediate threat to the plaintiff. The tortfeasor must have the apparent ability to carry out the harm. For instance, if a fist is shown from a distance that makes it impossible to execute the threat, then it does not constitute assault.

In most cases, Assault (which is mental) precedes Battery (which is physical).

c) False Imprisonment:

  • False imprisonment involves the imposition of complete restraint on an individual's liberty for a limited period and without any lawful justification.
  • It occurs when a person's freedom is unjustly taken away by confining them within four walls or by preventing them from leaving a specific location. Essential Elements of False Imprisonment Include: a) Total Restraint on an Individual's Liberty:
  • In criminal law, both total and partial restraint can be actionable offenses. However, in tort law, only total restraint is punishable.
  • False imprisonment occurs when a person is entirely deprived of their freedom to move beyond a specified limit, rather than simply being prevented from going in a particular direction while remaining free to choose another path.

Read moreShort tricks to write answers in Judiciary Mains Exam

b) Means of Escape:

  • False imprisonment may not apply if there is a viable means of escape. However, the means of escape should be comprehensible to the person who is detained. For instance, if the detained person is blind, they might not be able to locate the means of escape.
  • Additionally, the means of escape should be reasonable. If, for example, the window offering an escape route is too high and could cause injury to the detained person, then it may still be considered false imprisonment.

Defamation:

  • Defamation involves causing harm to an individual's reputation, goodwill, and character.
  • A person's reputation is regarded as a valuable asset, and anyone who injures another person's reputation does so at their own risk.
  • Defamation can be categorized into two types: i. Libel: This form of defamation occurs through representations made in a permanent form, such as in writing, printing, or pictures. ii. Slander: Slander pertains to the publication of defamatory statements in a transient form, such as spoken words or gestures.

Essential Elements of Defamation Include:

a) The Statement Must Be Defamatory:

A defamatory statement is one that tends to harm the reputation of the plaintiff.  If a statement causes someone to be regarded with feelings of hatred, contempt, ridicule, fear, dislike, or disesteem, it is considered defamatory. For example: A local newspaper published a false statement claiming that 'A' had eloped with 'C' the previous night. 'A' came from a well-educated family and was also a final-year BCom student. The news item was published irresponsibly and without any justification. This publication had a detrimental impact on 'A's' marriage prospects, and the statement was deemed defamatory, leading to liability for the defendants.

Trespass:

  • Trespass entails an unreasonable interference with property, land, a person, or goods.
  • In trespass, the legal rights of the owner are violated as they are deprived of their ability to enjoy the benefits of their property due to misappropriation or exploitation.
  • Trespass can be categorized into three types: i. Trespass to Person ii. Trespass to Land iii. Trespass to Goods

I. Trespass to Person: Occurs when there is interference or harm to a person's body, such as assault, battery, or false imprisonment.

  • It infringes upon a person's right to freedom.
  • It includes torts such as assault, battery, and false imprisonment. Example: Slapping someone or pointing a gun at them.

II. Trespass to Land: Involves interference with the possession of land without lawful justification.

  • Trespass can occur when a person enters another person's land themselves or through the use of some tangible object.
  • Going beyond the intended purpose for entering specific premises or crossing a boundary where one lacks authorization constitutes trespass.
  • If a person enters another's premises solely to save someone's life, it may not be considered trespass.

III. Trespass to Goods: Entails interference with goods in the plaintiff's possession without lawful justification.

  • Goods refer to personal property, whether movable or immovable, except for land.
  • Mistaken ownership cannot be used as a defense. For instance, if two friends are studying together and one mistakenly takes the other's book, this mistake of ownership cannot be a defense.
  • If the defendant wrongfully withholds goods belonging to the plaintiff and refuses to return them upon lawful demand, the plaintiff can recover them through an action for detinue.
  • If a person willfully and without any justification converts the goods of another person who is entitled to immediate use and possession of the goods, then the person is liable for conversion of chattel.

Read moreSimple tricks to enhance your English preparation for Judiciary Exams

Nuisance:

Nuisance refers to the unlawful interference with a person's use or enjoyment of land. Acts that interfere with comfort, health, safety, and so on are examples of nuisance. Interference can occur in various forms, such as noise, vibrations, smoke, excavation, and more.

Nuisance differs from trespass in several ways:

i. In trespass, interference is direct, whereas in nuisance, it is consequential.

ii. Trespass always involves physical or tangible objects, while nuisance can involve both tangible and intangible factors, like smoke or vibrations.

iii. Trespass is actionable per se, meaning the plaintiff doesn't need to prove actual damage, while in nuisance, special damage must be demonstrated.

Read About:  Judiciary Exam Preparation Tips

Essential elements of Nuisance:

a) Unreasonable interference:

  • Interference may result in damage to the plaintiff's property or personal discomfort in the enjoyment of the property.
  • Not all interference qualifies as a nuisance. For instance, if I own a house by the side of a road, I cannot bring a lawsuit for the inconvenience that is necessarily incidental to the traffic on the road.
  • An unreasonable activity cannot be justified simply by claiming that reasonable care was taken to prevent it from becoming a nuisance.
  • An act that is reasonable does not become unreasonable and actionable solely because the plaintiff is overly sensitive to it. For example, if noise from a neighbor doesn't bother an average person but disturbs the plaintiff due to their heightened sensitivity, it may not be considered a nuisance.

b) Interference with the enjoyment of land:

  • Interference may result in either (a) damage to property or (b) disruption of comfort or health.
  • Injury to property occurs when there is an unauthorized interference with the use of another person's property through tangible or intangible means, such as allowing tree branches to overhang onto another person's land.
  • Injury to comfort or health arises when there is interference with the comfort and convenience of using the premises. However, the law does not address trivial matters.

[Difference between Nuisance and Trespass – Nuisance may be caused by indirect physical interference, while Trespass always involves direct physical interference by the defendant.

For example: • If A throws a stone into B's house, it is considered Nuisance because the interference is indirect (by the stone), and A did not enter the premises. On the other hand, if A enters B's house without permission and without lawful justification, it would be considered Trespass because the interference is direct (resulting from the defendant's entry).]

Know more: How to prepare for judiciary exams 

Negligence:

  • Negligence refers to the absence of reasonable care that is expected from all individuals to prevent harm to others.
  • The burden of proof typically rests on the plaintiff to demonstrate that the defendant was likely negligent.

For example: Rohan, the owner of a large dog, left his dog unattended, and it attacked a passerby, causing severe injuries. In this case, it can be argued that Rohan's actions were negligent.

Essential elements of malicious prosecution:

a) Duty of care: The defendant must owe a duty of care to the plaintiff.

  • The plaintiff needs to establish that the defendant had a specific legal duty to exercise care and that this duty was breached.
  • Sometimes, the relationship between the plaintiff and defendant automatically gives rise to a legal duty or obligation to act in a certain manner. For instance, a doctor owes a duty of care to treat the patient appropriately.

b) Breach of duty: The defendant has breached the duty.

The defendant can be found negligent if they were aware that their actions could harm another person but did not act appropriately. For instance, if the defendant knows that their dog is aggressive, they should post a warning sign outside their house that says "Beware of dog." Failure to do so may be considered negligence.

c) Damage: The plaintiff must have suffered harm as a result of the defendant's actions. It is crucial to prove that the defendant's breach of duty directly caused harm to the plaintiff. The court has the discretion to assess the extent of the damage.

Nervous shock:

  • Nervous shock refers to when a person experiences physical injury as a result of a mental shock caused by something they have seen or heard.
  • Nervous shock is a psychological illness or injury that occurs due to the negligence of another person.
  • It occurs when there is a reasonable fear of immediate personal harm to oneself.

Get Details: Books for Judiciary Mains Exams 

For Example:

A person (A) falsely informed B's pregnant wife (C) that her husband (B) had suffered a nervous shock and had a serious accident resulting in both legs being broken. In this case, A was held responsible for causing nervous shock.

Questions from Law of Torts for Judiciary Exams 2023

Here are a few essential questions based on the law of torts for the Judicial Services Exam. Try answering these questions to check your preparation levels. Also, solving the previous year's Judiciary exam question papers will help understand the exam's difficulty level. 

Q1. Winfield defined 'tort' as

(a) a civil wrong for which the remedy is a common-law action for unliquidated damages and which is not exclusively the breach of trust or other merely eq­uitable obligation

(b) tortious liability arises from the breach of a duty primarily fixed by the law to­wards the persons generally, and its violation is redressible by an action for unliquidated damages

(c) an infringement of a right in rem of a private individual giving a right of compensation at the suit of the injured party

(d) none of the above.

Ans. Option b.

Q2. Tort is redressible by an action

(a) for restoration of the original position

(b) for unliquidated damages

(c) for liquidated damages

(d) all of the above.

Ans. Option b.

Q3. The 'tort of deceit' owes its origin to

(a) Pasley v. Freeman, (1789) 3 TR 51

(b) Lumley v. Gye, (1853) 2 E&B 216

(c) Rylands v. Fletcher, (1868) LR 3 HL 330

(d) Winsmore v. Greenbank, (1745) Willes 577.

Ans. Option a.

Q4. The 'tort of inducement a breach of contract' finds its origin in

(a) Lumley v. Gye

(b) Rookes v. Barnard

(c) Donoghue v. Stevenson

(d) Rylands v. Fletcher.

Ans. Option a.

Q5. The rule of 'strict liability is based on the decision in

(a) Donoghue v. Stevenson

(b) Rylands v. Fletcher

(c) Lumley v. Gye

(d) Champman v. Pickersgill.

Ans. Option b.

Q6. The tort is a violation of

(a) a right in personam

(b) a right in rem

(c) both right in personam & a right in rem

(d) neither a right in personam nor a right in rem.

Ans. Option b

Q7. Maxim injuria sine damno means

(a)violation of a legal right without any damage

(b) violation of a lawful request with damage

(c) damage without violation of the legal right

(d) no damage & no breach of legal rights.

Ans. Option a.

Learn more: Judiciary Exam 2023 Online Coaching

Q8. Malice in law means

(a) a wrongful act was done intentionally but without just cause or excuse

(b) a wrongful act was done intentionally with just cause & excuse

(c) a wrongful act was done intentionally with a good motive

(d) a wrongful act was done intentionally with an evil motive.

Ans. Option a.

Q9. The defence of volenti non-fit injuria, is not available

(a) if the consent is obtained by compul­sion

(b) if the consent is obtained by fraud

(c) if the consent is obtained under a mis­take

(d) all of the above.

Ans. Option d.

Q10. Inevitable accident means

(a) an act of God

(b) an unexpected injury which could have been foreseen & avoided

(c) unforeseen damage which could not have been expected & avoided

(d) both (a) & (b).

Ans. Option c.

Q11. Act of State

(a) cannot be between a sovereign and the subjects of another State

(b) cannot be between a ruler and his subjects

(c) both (a) & (b)

(d) neither (a) nor (b).

Ans. Option a.

Q12. The liability of joint tortfeasors is

(a) only joint

(b) only several

(c) joint & several

(d) neither joint nor several.

Ans. Option c.

Download free: Judiciary Exams Question Paper 

Q13. In the case of independent tort features

(a) action against one bar the action against the others

(b) action against some bars the action against the others

(c) action against one does not bar action against the other

(d) both (a) & (b).

Ans. Option b.

Q14. Vicarious liability commensurates with that of

(a) independent tort feasors

(b) joint tort feasors

(c) composite tort feasors

(d) both (a) & (c).

Ans. Option b.

Q15. For vicarious liability, a ser­vant is a person

(a) on whom the master has command of what to do & how to do

(b) on whom the master has a command of what to do

(c) whom the master can hire & fire

(d) only (a) & (c) above.

Ans. Option d.

Q16. Vicarious liability of master arises

(a) in case of theft of property by a third person, bailed to the master

(b) in case of theft of property of a third person not bailed to the master

(c) neither (a) nor (b) as the act of commit­ting theft is not covered

(d) Both (a) and (b) as it makes no differ­ence whether the goods are bailed or not bailed to the master.

Ans. Option b.

Q17. The doctrine of common employment means

(a) if the wrongdoer servant and the per­-son injured are fellow servants, the master is not liable

(b) if the wrongdoer servant and the per-­son injured are fellow servants, the master is liable

(c) if the wrongdoer servant and the per-­son injured are not fellow servants, the master is not liable

(d) none of the above.

Ans. Option a.

Also Read: Judiciary Interview Preparation Strategies & Tricks

Q18. In India, the scope of the doctrine of com­mon employment was restricted by

(a) The Indian Employers' Liability Act, 1938

(b) The Workmen's Compensation Act, 1923

(c) The Employees' State Insurance Act, 1948

(d) all of the above.

Ans. Option a.

Q19. ï»¿Remoteness of damages is determined by

(a) the test of directness

(b) the test of reasonable foresight

(c) both (a) & (b)

(d) none of the above.

Ans. Option b.

Q20. The test of reasonable foresight in deter­mining the remoteness of damages was first applied in

(a) Re: Polerris

(b) Wagon Mannd case

(c) Doughty v. Turner Manufacturing Co. Ltd.

(d) SCM (United Kingdom) Ltd. v. W.J. Whittal & Sons.

Ans. Option b.

Q21. For 'false imprisonment', there should be

(a) total restraint on the liberty of a person

(b) a partial restraint on the liberty of a person

(c) means of escape

(d) all of the above.

Ans. Option a.

Ace the exam with the best PCS J online coaching classes.

Conclusion:

For Torts an aspirtant must start their preparation for torts as soon as you decide that you are preparing for a state that covers torts in their syllabus. Though the concepts seem easy in torts but it is important to start its preapration from the starting only and keep revising it on regular basis.

All the best Aspirants!

Frequently Asked Questions

What are the stages in the Delhi Judiciary Exam Selection Process?

Expand Faq Icon

Which are important topics in Judiciary Prelims Exam?

Expand Faq Icon

What is the marking scheme of Judiciary Prelims and Mains Exam?

Expand Faq Icon

Is Viva-Voce stage compulsory?

Expand Faq Icon

What is the marking scheme of Judiciary Viva-Voce stage?

Expand Faq Icon

Do really solving Judiciary prelims Question Papers will help score better in the exam?

Expand Faq Icon

How to download Delhi judiciary Civil Law previous year question papers?

Expand Faq Icon

How to download Haryana judiciary General Knowledge previous year question papers?

Expand Faq Icon

ABOUT TOP RANKERS

Toprankers, launched in 2016, is India’s most preferred digital counselling & preparation platform for careers beyond engineering & medicine. We envision to build awareness and increase the success rate for lucrative career options after 12th. We offer best learning practices and end-to-end support to every student preparing for management, humanities, law, judiciary & design entrances.

E

: support@toprankers.com

P

: +91-7676564400

Social Channels

App Badge

Google Play Icon
Chat to Toprankers Team